由买买提看人间百态

boards

本页内容为未名空间相应帖子的节选和存档,一周内的贴子最多显示50字,超过一周显示500字 访问原贴
Quant版 - GS一道题,转自glassdoor。
相关主题
出个好玩的题吧求一个Entry level quant的内推,内附详细背景~
ito积分问一个sampling from multivariate distribution 的问题
BulletTooth的题讨论[合集] 贡献个题目(SDE)
Monte Carlo算出来的VaR总是比parametric算出来的大?请教两个portfolio合并的VaR的问题
Asset allocation 问题请教面试题目,Stochastic calculus求教
JP 2nd round 电面问一个stochastic calculus问题
贴个认真内推职位a random variable question
求 market risk modeling 面试资料请教 chimbo's two interview questions
相关话题的讨论汇总
话题: var话题: variance话题: bound话题: give
进入Quant版参与讨论
1 (共1页)
l*********t
发帖数: 89
1
What does a distribution with a maximal variance look like which is only
defined between 0 and 1? Give the proof.
t*******e
发帖数: 172
2
I can not help present the following idea.
It is easy to find two point distribution with probability 1/2 give us the
variance 1/4.
Then, we wanna prove it is an upper bound, I am not sure weather this method
was mentioned in this webpage.
Let us do the Monte Carlo Simulation, given a distribution p(x), we take n
samples from it. Then the variance is estimated by
(n(X_1^2+...+X_n^2)-(X_1+..+X_2)^2)/n^2, which is no more than 1/4, let n
tend to infinity, we know the variance no more than 1/4. This proves the
upper bound is 1/4.
l*********t
发帖数: 89
3
Cool idea. Thank you!

method

【在 t*******e 的大作中提到】
: I can not help present the following idea.
: It is easy to find two point distribution with probability 1/2 give us the
: variance 1/4.
: Then, we wanna prove it is an upper bound, I am not sure weather this method
: was mentioned in this webpage.
: Let us do the Monte Carlo Simulation, given a distribution p(x), we take n
: samples from it. Then the variance is estimated by
: (n(X_1^2+...+X_n^2)-(X_1+..+X_2)^2)/n^2, which is no more than 1/4, let n
: tend to infinity, we know the variance no more than 1/4. This proves the
: upper bound is 1/4.

xf
发帖数: 68
4
Just start with definition
For any rv X and real number a, Var(X)<=E(X-a)^2.
Set a=1/2. when X in[0,1]
then Var(X)<=E(1/2)^2=1/4
c******s
发帖数: 270
5
Var(x) = E[x^2] - E[x]^2
<= E[x] - E[x]^2
= 1/4 - (1/2 - E[x])^2
<= 1/4
when E[x^2] = E[x] =1/2, it reaches the higher bound
1 (共1页)
进入Quant版参与讨论
相关主题
请教 chimbo's two interview questionsAsset allocation 问题请教
面经 -- RBC & CitiJP 2nd round 电面
答谢文,谈谈VIX 和 VIX future贴个认真内推职位
who can prove the mean(R) and Var(R) of runs test?求 market risk modeling 面试资料
出个好玩的题吧求一个Entry level quant的内推,内附详细背景~
ito积分问一个sampling from multivariate distribution 的问题
BulletTooth的题讨论[合集] 贡献个题目(SDE)
Monte Carlo算出来的VaR总是比parametric算出来的大?请教两个portfolio合并的VaR的问题
相关话题的讨论汇总
话题: var话题: variance话题: bound话题: give